Search found 26 matches


OA is A. I chose E because I felt that the conclusion was "Hunters are not alone to blame." That bears may be to blame seemed like more of a subconclusion, which wasn't an answer. The problem with choosing E is that it is clearly referring to last sentence. Since it was from GMAT Prep, no ...

by cm47323

Thu Apr 16, 2009 9:13 pm
Forum: Critical Reasoning
Topic: Boldface from GMATPrep
Replies: 9
Views: 3767

The correct answer is C. This is a classic Bayesian probability issue. If you recognize this problem as such, you'll prephrase the correct answer and be done. If not, try process of elimination: A. no value judgment B. tougher to eliminate. in this case, this error in reasoning does not apply since ...

by cm47323

Thu Apr 16, 2009 12:59 am
Forum: Critical Reasoning
Topic: Cocaine
Replies: 3
Views: 2338

Boldface from GMATPrep

Hunter: Hunters alone are blamed for the decline in Greenrock National Forest's deer population over the past ten years. Yet clearly, black bears have also played an important role in this decline. In the past ten years, the forest's protected black bear population has risen sharply, and examination...

by cm47323

Wed Apr 15, 2009 8:40 pm
Forum: Critical Reasoning
Topic: Boldface from GMATPrep
Replies: 9
Views: 3767

haha, i need to read more carefully... makes a big difference

by cm47323

Sun Apr 05, 2009 1:03 am
Forum: Problem Solving
Topic: a video game cartridge
Replies: 3
Views: 1726

100,000=3000/(2*10+a)
20+a=.03
a=-19.97

Plug 20 into the function and the answer is ~ 150. I think the function should not have a plus sign in the denominator. Either that or I'm way off base

by cm47323

Sat Apr 04, 2009 1:40 pm
Forum: Problem Solving
Topic: a video game cartridge
Replies: 3
Views: 1726

Unless I missed some way to simplify the math, this problem isn't very GMATy. First need to take sqrt(1256/pi) to get the radius. You can guesstimate to around 20. Then need the circumference which involves multiplying by pi and then dividing the distance by the rate. I get 75 as well, but again, I ...

by cm47323

Sat Apr 04, 2009 1:28 pm
Forum: Problem Solving
Topic: Car A and Car B
Replies: 10
Views: 1997

Area = 1/2(bh) = 12
.5(4)h=12
h=6

so the absolute value of the y-intercept is 6. We can't know it is positive or negative unless we know more about line k. Since we know it has a positive x-intercept and is positively sloped, its y-intercept is negative.

A

by cm47323

Tue Mar 31, 2009 7:00 pm
Forum: Problem Solving
Topic: xy plane
Replies: 8
Views: 6303

Now the correct answer is 1/8 i.e. answer choice D. Also how u've considered CE and CA on a single straight line EA. Please elaborate the solution. No, it isn't. What's the source? While GMAT diagrams are not necessarily to scale, this is an abomination. There is no line segment DE. Fwiw, I solved ...

by cm47323

Tue Mar 31, 2009 3:07 pm
Forum: Problem Solving
Topic: Figure
Replies: 12
Views: 12183

scoobydooby wrote: probabilty of no girls being together: 10!-8!*3!/10!
You solved P(girls not all together), different from P(no girls together). This question was poorly worded from the start...

by cm47323

Mon Mar 30, 2009 12:38 pm
Forum: Problem Solving
Topic: Probability boys girls
Replies: 10
Views: 2251

If x is negative, y = abs(x) + x = 0, one x is positive, the other negative
If x is positive, y = abs (x) + x = 2x

By this logic, Y can never be negative. It has nothing to do with the information in (2). 2 just mandates that y=0

by cm47323

Mon Mar 30, 2009 11:00 am
Forum: Data Sufficiency
Topic: Integers
Replies: 4
Views: 1229

maihuna, you're right. i stand corrected - E

by cm47323

Mon Mar 30, 2009 10:51 am
Forum: Data Sufficiency
Topic: Three digit Integer
Replies: 3
Views: 981

because 8/9+1/8=73/72, so X +Y < 73/72, which doesn't answer anything

by cm47323

Mon Mar 30, 2009 10:35 am
Forum: Data Sufficiency
Topic: X,Y Inequality
Replies: 2
Views: 1256

This one is kinda tricky. It does not say that K has a total of 4 factors, which would make it nonprime. 4 different prime factors can mean something like K= 2^2 * 3^2 * 5^2 * 7^2 sqrt(K) = 2*3*5*7 As long as all of the powers are even, it's going to be a square. If they aren't all even, it won't be...

by cm47323

Mon Mar 30, 2009 10:33 am
Forum: Data Sufficiency
Topic: Prime Number
Replies: 1
Views: 930

(1) K + 150 = 4xx 250<K<349. Insufficient (2) K + 25 = x7x. Insufficient Combined. If we know the tens digit is a 7. this means that K must have a 2 as hundreds digit. There is no way to get a 7 in the tens digit when K is in the 300s since K<349. Therefore, hundreds digit is 2. Sufficient C

by cm47323

Mon Mar 30, 2009 10:22 am
Forum: Data Sufficiency
Topic: Three digit Integer
Replies: 3
Views: 981

We have two possibilites If x is negative, y = abs(x) + x = 0, one x is positive, the other negative If x is positive, y = abs (x) + x = 2x (1) x < 0. y=0. logic above, Sufficient (2) y < 1. This tripped me up initially, but the key is that y is an integer, so y<=0. Since y can't be negative. y=0. S...

by cm47323

Mon Mar 30, 2009 9:47 am
Forum: Data Sufficiency
Topic: Integers
Replies: 4
Views: 1229